Question and Answers Forum

All Questions      Topic List

Algebra Questions

Previous in All Question      Next in All Question      

Previous in Algebra      Next in Algebra      

Question Number 148522 by mathdanisur last updated on 28/Jul/21

Answered by Rasheed.Sindhi last updated on 29/Jul/21

P(x)=16x^2 +24x+9  Q(x)=x^3 −3x^2 +2x+2  If x=−1.25  ((P^3 +Q^3 )/(P^2 −PQ+Q^2 ))+((P^3 −Q^3 )/(P^2 +PQ+Q^2 ))=?   ((P^3 +Q^3 )/(P^2 −PQ+Q^2 ))+((P^3 −Q^3 )/(P^2 +PQ+Q^2 ))  =(P+Q)+(P−Q)  =2P=2(16x^2 +24x+9)       =2(16(−(5/4))^2 +24(−(5/4))+9)       =2(16(((25)/(16)))−6(5)+9)       =2(25−30+9)=2(4)=8

$${P}\left({x}\right)=\mathrm{16}{x}^{\mathrm{2}} +\mathrm{24}{x}+\mathrm{9} \\ $$$${Q}\left({x}\right)={x}^{\mathrm{3}} −\mathrm{3}{x}^{\mathrm{2}} +\mathrm{2}{x}+\mathrm{2} \\ $$$${If}\:{x}=−\mathrm{1}.\mathrm{25} \\ $$$$\frac{{P}^{\mathrm{3}} +{Q}^{\mathrm{3}} }{{P}^{\mathrm{2}} −{PQ}+{Q}^{\mathrm{2}} }+\frac{{P}^{\mathrm{3}} −{Q}^{\mathrm{3}} }{{P}^{\mathrm{2}} +{PQ}+{Q}^{\mathrm{2}} }=?\: \\ $$$$\frac{{P}^{\mathrm{3}} +{Q}^{\mathrm{3}} }{{P}^{\mathrm{2}} −{PQ}+{Q}^{\mathrm{2}} }+\frac{{P}^{\mathrm{3}} −{Q}^{\mathrm{3}} }{{P}^{\mathrm{2}} +{PQ}+{Q}^{\mathrm{2}} } \\ $$$$=\left({P}+{Q}\right)+\left({P}−{Q}\right) \\ $$$$=\mathrm{2}{P}=\mathrm{2}\left(\mathrm{16}{x}^{\mathrm{2}} +\mathrm{24}{x}+\mathrm{9}\right) \\ $$$$\:\:\:\:\:=\mathrm{2}\left(\mathrm{16}\left(−\frac{\mathrm{5}}{\mathrm{4}}\right)^{\mathrm{2}} +\mathrm{24}\left(−\frac{\mathrm{5}}{\mathrm{4}}\right)+\mathrm{9}\right) \\ $$$$\:\:\:\:\:=\mathrm{2}\left(\mathrm{16}\left(\frac{\mathrm{25}}{\mathrm{16}}\right)−\mathrm{6}\left(\mathrm{5}\right)+\mathrm{9}\right) \\ $$$$\:\:\:\:\:=\mathrm{2}\left(\mathrm{25}−\mathrm{30}+\mathrm{9}\right)=\mathrm{2}\left(\mathrm{4}\right)=\mathrm{8} \\ $$$$ \\ $$

Commented by mathdanisur last updated on 28/Jul/21

Cool Ser, thank you

$${Cool}\:{Ser},\:{thank}\:{you} \\ $$

Answered by Olaf_Thorendsen last updated on 28/Jul/21

R = ((P^3 +Q^3 )/(P^3 −PQ+Q^2 ))+((P^3 −Q^3 )/(P^3 +PQ+Q^2 ))  R = (((P+Q)(P^2 −PQ+Q^2 ))/(P^3 −PQ+Q^2 ))+(((P−Q)(P^2 +PQ+Q^2 ))/(P^3 +PQ+Q^2 ))  R = P+Q+P−Q = 2P  R(−(5/4)) = 2(16.((25)/(16))−24.(5/4)+9)  R(−(5/4)) = 8

$$\mathrm{R}\:=\:\frac{{P}^{\mathrm{3}} +{Q}^{\mathrm{3}} }{{P}^{\mathrm{3}} −{PQ}+{Q}^{\mathrm{2}} }+\frac{{P}^{\mathrm{3}} −{Q}^{\mathrm{3}} }{{P}^{\mathrm{3}} +{PQ}+{Q}^{\mathrm{2}} } \\ $$$$\mathrm{R}\:=\:\frac{\left({P}+{Q}\right)\left({P}^{\mathrm{2}} −{PQ}+{Q}^{\mathrm{2}} \right)}{{P}^{\mathrm{3}} −{PQ}+{Q}^{\mathrm{2}} }+\frac{\left({P}−{Q}\right)\left({P}^{\mathrm{2}} +{PQ}+{Q}^{\mathrm{2}} \right)}{{P}^{\mathrm{3}} +{PQ}+{Q}^{\mathrm{2}} } \\ $$$$\mathrm{R}\:=\:{P}+{Q}+{P}−{Q}\:=\:\mathrm{2}{P} \\ $$$${R}\left(−\frac{\mathrm{5}}{\mathrm{4}}\right)\:=\:\mathrm{2}\left(\mathrm{16}.\frac{\mathrm{25}}{\mathrm{16}}−\mathrm{24}.\frac{\mathrm{5}}{\mathrm{4}}+\mathrm{9}\right) \\ $$$${R}\left(−\frac{\mathrm{5}}{\mathrm{4}}\right)\:=\:\mathrm{8} \\ $$

Commented by mathdanisur last updated on 28/Jul/21

Cool Ser, thank you

$${Cool}\:{Ser},\:{thank}\:{you} \\ $$

Terms of Service

Privacy Policy

Contact: info@tinkutara.com